Đến nội dung

VIF

VIF

Đăng ký: 21-04-2009
Offline Đăng nhập: 29-09-2013 - 20:37
-----

Đề thi chọn học sinh giỏi Phú Yên 2009

03-12-2009 - 20:20

SỞ GIÁO DỤC VÀ ĐÀO TẠO PHÚ YÊN
KÌ THI CHỌN HỌC SINH GIỎI CẤP TỈNH LỚP 12 THPT
NĂM HỌC 2009-2010
Môn thi: TOÁN
Thời gian: 180 phút
(Không kể thời gian giao đề)
****



Câu 1. (5 điểm) Cho hàm số $y = \dfrac{{2x}}{{x - 1}}$ có đồ thị ©.
a) Khảo sát và vẽ đồ thị hàm số ©, từ đó suy ra đồ thị $\left( {{C_1}} \right)$ của hàm số $y = \dfrac{{\left| x \right|}}{{\left| x \right| - 1}}$.
b) Dựa vào đô thị$({C_1})$, biện luận theo tham số $m$ số nghiệm $x$ thuộc đoạn $\left[ { - 1;2} \right]$ của phương trình: $\left( {m - 2} \right)\left| x \right| - m = 0$

Câu 2. (5 điểm)
a) Giải phương trình: $\left( {x - 1} \right)\left( {x + 2} \right) - \left( {x - 1} \right)\sqrt {\dfrac{{x + 2}}{{x - 1}}} - 2 = 0$
b) Tìm x,y,z biết
$\left\{ {\begin{array} {\sqrt x + \sqrt y + \sqrt z = 3} \\ {\left( {1 + x} \right)\left( {1 + y} \right)\left( {1 + z} \right) = {{\left( {1 + \sqrt[3]{{xyz}}} \right)}^3}} \\\end{array}} \right.$

Câu 3. (5 điểm) Cho hình vuông $ABCD$. Trên các cạnh CB và CD lần lượt lấy các điểm E,F sao cho $\dfrac{{BE}}{{BC}} = k$ và$\dfrac{{DF}}{{DC}} = \dfrac{{1 - k}}{{1 + k}}$, với$0 < k < 1$. Đoạn thẳng BD cắt AE và AF tại H và G tương ứng. Đường vuông góc với EF kẻ từ A cắt BD tại P. Chứng minh rằng

$\dfrac{{PG}}{{PH}} = \dfrac{{DG}}{{BH}}$


Câu 4. (3 điểm) Trong mặt phẳng (P) cho điểm O cố định và d là đường thẳng quay quanh O. Lấy S ngoài (P) có hình chiếu vuông góc trên (P) là H, với $H \ne O$. Qua S dựng đường vuông góc với mặt phẳng xác định bởi S và d. Đường thẳng này cắt (P) tại N. Tìm quỹ tích điểm N khi d thay đổi.

Câu 5. (2 điểm) Cho đa thức : $f\left( x \right) = {x^4} - 2011{x^3} + \left( {2010 + a} \right){x^2} - 2009 + a$, với a là số nguyên. Chứng minh rằng đa thức không thể có hai nghiệm nguyên (phân biệt hay trùng nhau).

-HẾT-


ddbđtVN 2009 VÀ CUỘC THI VICFJ

15-11-2009 - 16:47

Mở đầu, tôi xin viết ra đây một bài toán bất đẳng thức:
Bài toán (VQBC). Cho các số thực dương $a,b,c$ sao cho $a + b + c = 1$. Chứng minh rằng
$\dfrac {{36}}{{{a^2}b + {b^2}c + {c^2}a}} + \dfrac {1}{{abc}} \ge 343$

Đây là một bài toán rất nghệ thuật, rất chặt chẽ với dấu đẳng thức không xảy ra tại tâm $a: b: c = {\sin ^2}\dfrac {{4\pi }}{7}: {\sin ^2}\dfrac {{2\pi }}{7}: {\sin ^2}\dfrac {\pi }{7}$, cụ thể hơn là $a = \dfrac {4}{7}{\sin ^2}\dfrac {{4\pi }}{7},b = \dfrac {4}{7}{\sin ^2}\dfrac {{2\pi }}{7},c = \dfrac {4}{7}{\sin ^2}\dfrac {\pi }{7}$. Các bạn có thể tham gia thảo luận thêm về bất đẳng thức này tại đây

Bây giở thì bạn đã hiểu tại sao bất đẳng thức lại được các bạn học sinh VN quan tâm nhiều nhất. giáo sư Hoàng Tụy đã từng nhắc nhở các bạn trẻ yêu Toán rằng: Các nhà Toán học thường làm việc với bất đẳng thức nhiều hơn đẳng thức. chính vì lí do đó, một diễn đàn chỉ chuyên về bất đẳng thức đã được thành lập với tên miền chính thức là http://www.ddbdt.co.cc (VIMF - The VietNam Inequality Mathematic Forum). Ngoài hệ thống diễn đàn (forum) để thảo luận các vấn đề thời sự bất đẳng thức , chúng tôi còn đưa hệ thống trang chủ vào cùng hoạt động nhằm post lên những bài toán hay, đẹp và lạ (như tin nóng đăng trên trang nhất tờ bào vậy), để khích lệ các thành viên. Chúng tôi thật sự vui mừng khi thống kê trong vòng 1 tháng kể từ khi diễn đàn hoạt động với những con số ấn tượng:

Ðề tài: 208, Bài gởi: 834, Thành viên: 86, Thành viên kích hoạt: 85


Điều đó nói lên rằng DĐBĐTVN thật sự là một sân chơi bổ ích cho các bạn yêu thích bất đẳng thức.. Và đề nâng cao tinh thần đó, chúng tôi quyết định tổ chức thêm một cuộc thi bất đẳng thức dành cho các bạn từ lớp 8 đến lớp 11 (VICFJ), sau khi đã tổ chức thành công cuộc thi bất đẳng thức VIC làn thứ I. Cho tiết và đăng kí tham gia cuộc thi VICFJ (The VietNam Inequality Contest For Junior) tại đây

Kì thi olympic bất đẳng thức vic 2009

23-06-2009 - 21:10

Trong thời gian vừa qua, DDBDTVN đã nhận được sự quan tâm ủng hộ của các bạn. Tuy nhiên không ít bạn phàn nàn về diễn đàn như: tên miền quá dài, thiếu một số chức năng và loading vào forum chậm, đôi khi loading không vào được forum,.... Vâng, chúng tôi rất chân thành cảm ơn các ý kiến của các bạn và để khắc phục tất cả những điều đó, chúng tôi đã cố gắng hoàn thành sớm phiên bản 2 năm 2009 của DDBDTVN và đến này đã hoàn tất. Nay chúng tôi xin thông báo chính thức đến các bạn, DDBDTVN chuyển về http://www.vimf.co.cc (ngắn gọn), các chức năng up file, avatar đã khá đầy đủ, forum loading nhanh và không xảyra tình trangjloading không vào được forum nên các bạn có thể yên tâm với mỗi bài viết của mình,.... Nhìn chung, forum VIMF đã khá giống với Mathlink (chưa thật sự giống lắm). Một lần nữa xin gửi lời cảm ơn đến các bạn và mong các bạn tiếp tục ủng hộ DDBDTVN phiên bản 2 http://www.vimf.co.cc.

ĐỀ THI VIC VÒNG 1


Bài 1. Cho các số thực không âm $a,b,c$ sao cho $ab + bc + ca = 1$. Chứng minh rằng

$\dfrac{{3ab + 1}}{{a + b}} + \dfrac{{3bc + 1}}{{b + c}} + \dfrac{{3ca + 1}}{{c + a}} \ge 4$

Bài 2. Cho các số thực không âm a,b,c sao cho $a \ge b \ge c$. Chứng minh rằng

$\dfrac{{{a^3}b}}{{{b^2} - bc + {c^2}}} + \dfrac{{{b^3}c}}{{{c^2} - ca + {a^2}}} + \dfrac{{{c^3}a}}{{{a^2} - ab + {b^2}}} \ge ab + bc + ca$


Bài 3. Cho các số thực $x,y,z$ sao cho $xyz = 1$. Chứng minh rằng

$\dfrac{{3 - x}}{{{{\left( {1 + x} \right)}^2}}} + \dfrac{{3 - y}}{{{{\left( {1 + y} \right)}^2}}} + \dfrac{{3 - z}}{{{{\left( {1 + z} \right)}^2}}} + \dfrac{{16}}{{\left( {1 + x} \right)\left( {1 + y} \right)\left( {1 + z} \right)}} \ge \dfrac{{47}}{{16}}$

Bài 4. Cho các số thực phân biệt a,b,c và số thực bất kì k∈[0;1]. Chứng minh rằng

$\dfrac{{a\left( {a + kb} \right)}}{{{{\left( {a - b} \right)}^2}}} + \dfrac{{b\left( {b + kc} \right)}}{{{{\left( {b - c} \right)}^2}}} + \dfrac{{c\left( {c + ka} \right)}}{{{{\left( {c - a} \right)}^2}}} \ge \dfrac{7}{8}$

Bài 5. Cho $A, B, C$ là 3 góc của một tam giác nhọn. Tìm giá trị nhỏ nhất của

$2tanA+9tanB+17tanC$

Bài 6. Cho các số thực không âm a,b,c. Tìm giá trị nhỏ nhất của biểu thức

$P = \left( {ab + bc + ca} \right)\left( {\dfrac{1}{{{{\left( {2a - b} \right)}^2}}} + \dfrac{1}{{{{\left( {2b - c} \right)}^2}}} + \dfrac{1}{{{{\left( {2c - a} \right)}^2}}}} \right)$

Bài 7. Cho $x,y,z$ là độ dài 3 cạnh của một tam giác có diện tích $S$, và các dương $a,b,c$ sao cho $4xyz = {a^2}x + {b^2}y + {c^2}z + abc$. Chứng minh rằng

${a^2} + {b^2} + {c^2} \ge 4\sqrt 3 S$


S.O.C - Kĩ thuật phân tích bình phương cho bdt hoán vị

27-04-2009 - 13:48

DIEN DAN BAT DANG THUC VIET NAM

VietNam Inequality Mathematic Forum

▼▼▼▼▼
www.vnineqmath.tk

Tác Giả Bài Viết:

Admin

♦♦♦
Bài viết này (cùng với file đính kèm) được tạo ra vì mục đính giáo dục. Không được sử dụng bản ebook này dưới bất kì mọi mục đính thương mại nào, trừ khi được sự đồng ý của tác giả. Mọi chi tiết xin liên hệ: www.vnineqmath.tk



KĨ THUẬT PHÂN TÍCH BÌNH PHƯƠNG CHO BẤT ĐẲNG THỨC HOÁN VỊ
SUM OF CYCLIC (S.O.C)

I/ Lời nói đầu.

Bất đẳng thức hoán vị là những bài toán rất đẹp bới sự phát biểu đơn giản nhẹ nhàng của chúng. Tuy nhiên, việc giải chúng thì ngược lại, việc tìm một lời giải cho chúng vô cùng vất vả và khó khăn. Và đối với những bài toán có 2 đẳng thức trở lên thì mọi việc lại càng trở nên khó khăn hơn. Sau một thời gian học hỏi kinh nghiệm và tìm tòi, tôi đã tìm được một kĩ thuật để đánh giá cho những bất đẳng thức hoán vị đơn giản. Do độ khó của các bài toán nên đôi khi một số lời giải có đôi chút dài, nhưng bù lại là ta có thể làm chặt cho một số bài toán (đây là một điều bất ngờ mà kĩ thuật này mang lại).
Cũng xin nói thêm rằng: bất đẳng thức hiện đại rất phong phú với rất nhiều bài tập. Tuy nhiên với bất đẳng thức hoán vị vòng quanh thì khác, nó rất ít nên có thể coi là những bài toán hiếm. Việc tạo ra một bất đẳng thức đúng đã là khó mà để bất đẳng thức đó hay thì càng khó hơn, nên đối với bất đẳng thức hoán vị thì điều đó lại càng khó thực hiện. Vì thế kĩ thuật này chỉ là một công cụ nhỏ nhưng lại vô cùng hữu ích để các bạn có thêm một hướng giải quyết các bài toán bất đẳng thức hoán vị vòng quanh ba biến.
Mặc dù bài viết được hoàn thành trong lúc tôi đang cắm trại nên rất mệt, nhưng tôi vẫn cố gắng hoàn thành bài viết này trong một ngày trọng đại 26-3-2009 (Đoàn thanh niên Cộng sản Hồ Chí Minh). Vì thế tôi sẽ rất hoan nghênh những sự đóng góp, tìm tòi sáng tạo thêm cho kĩ thuật này từ phía các bạn. Mọi thắc mắc – đóng góp ý kiến xin vui long lien hệ theo địa chỉ:
E-mail: [email protected] hoặc YM: vnineq

Tác giả
VIF


II/ Cơ sở của kĩ thuật.

Sẽ thật bất ngờ nếu tôi nói với các bạn rằng cơ sở của kĩ thuật này là phương pháp phân tích bình phương S.O.S: là đưa bất đẳng thức thuần nhất ba biến a,b,c về dạng:
$S_a (b - c)^2 + S_b (c - a)^2 + S_c (a - b)^2 \ge 0$
Đối với bất đẳng thức đối xứng ba biến thì việc quy về dạng chính tắc S.O.S như trên là đơn giản giúp ta dễ dàng giải quyết bài toán. Tuy nhiên, đối với bất đẳng thức hoán vị vòng quanh thì cách quy trên đôi khi không thích hợp và tạo ra các hệ số $S_a ;S_b ;S_c$ rất cồng kềnh và khó xử lí. Trong trường hợp đó có một cách khác là quy về dạng: (Tôi tạm gọi nó là phân tích bình phương hoán vị S.O.C)
$S_a (b - c)^2 + S_b (c - a)^2 + S_c (a - b)^2 \ge S(a - b)(b - c)(c - a)$
Cách quy trên có gì lợi?:
- Thứ nhất: đối với các dạng hoán vị vòng quanh thì nó tự nhiên và đơn giản hơn cách đưa về S.O.S chính thống.
- Thứ hai: đối với bất đẳng thức hoán vị thì ta chỉ cần xét một trong 2 khả năng sau:
+ Một trong ba số là lớn nhất (giả sử $a = m{\rm{ax}}\{ a,b,c{\rm{\} }}$)thì ta xét 2 trường hợp có thể xảy ra là $a \ge b \ge c$ và $a \ge c \ge b$.
+ Một trong ba ở giữa 2 số kia (giả sử là b), thì ta xét 2 trường hợp có thể xảy ra là là $a \ge b \ge c và c \ge b \ge a$.
Vì vậy, nếu vế trái và S không âm thì ta chỉ xét trường hợp c \ge b \ge a mà bỏ qua trường hợp $a \ge b \ge c$.
Cuối cùng cũng xin lưu ý luôn là đối với các bài toán sau đây chúng ta cũng chỉ xét trường hợp $c \ge b \ge a$ (khi đó $(a - b)(b - c)(c - a) \ge 0 \Rightarrow a^2 b + b^2 c + c^2 a \le ab^2 + bc^2 + ca^2$), còn với trường hợp $a \ge b \ge c$ thì $S(a - b)(b - c)(c - a) \le 0$, và ta chỉ phải làm theo phương pháp truyền thống S.O.S là chứng minh bất đẳng thức:
$S_a (b - c)^2 + S_b (c - a)^2 + S_c (a - b)^2 \ge 0$

III/ Phân tích cơ sở.
1.$ab^2 + bc^2 + ca^2 - a^2 b - b^2 c - c^2 a = (a - b)(b - c)(c - a)$
2.$ab^2 + bc^2 + ca^2 - 3abc = \dfrac{1}{2}\left( {ab^2 + bc^2 + ca^2 - a^2 b - b^2 c - c^2 a + ab^2 + bc^2 + ca^2 + a^2 b + b^2 c + c^2 a - 6abc} \right) = \dfrac{1}{2}\left( {(a - b)(b - c)(c - a) + a(b - c)^2 + b(c - a)^2 + c(a - b)^2 } \right)$
3.$\dfrac{{a - b}}{{a + b}} + \dfrac{{b - c}}{{b + c}} + \dfrac{{c - a}}{{c + a}} = \dfrac{{ - (a - b)(b - c)(c - a)}}{{(a + b)(b + c)(c + a)}}$
4.$\dfrac{a}{{a + b}} + \dfrac{b}{{b + c}} + \dfrac{c}{{c + a}} = \dfrac{1}{2}\left( {\dfrac{{a + b + a - b}}{{a + b}} + \dfrac{{b + c + b - c}}{{b + c}} + \dfrac{{c + a + c - a}}{{c + a}}} \right) = \dfrac{1}{2}\left( {3 + \dfrac{{a - b}}{{a + b}} + \dfrac{{b - c}}{{b + c}} + \dfrac{{c - a}}{{c + a}}} \right) = \dfrac{1}{2}\left( {3 - \dfrac{{(a - b)(b - c)(c - a)}}{{(a + b)(b + c)(c + a)}}} \right)$

5.$ab^3 + bc^3 + ca^3 - a^3 b - b^3 c - c^3 a = (a + b + c)(a - b)(b - c)(c - a)$
Bên cạnh các phân tích sơ sở này còn rất nhiều cách phân tích khác mà các bạn có thể tự tìm thấy trong quá trình giải toán.


IV/ Xây dựng định lí.

Chúng ta sẽ xây dựng định lí, đưa ra các tiêu chuẩn từ cách phân tích
$S_a (b - c)^2 + S_b (c - a)^2 + S_c (a - b)^2 \ge S(a - b)(b - c)(c - a)$
Chú ý: ở đây ta chỉ xét đến trường hợp $c \ge b \ge a$
Như thế thì $(a - b)(b - c) \ge 0$ nên
$S_a (b - c)^2 + S_b (c - a)^2 + S_c (a - b)^2 = S_a (b - c)^2 + S_b (a - b + b - c)^2 + S_c (a - b)^2 = \left( {S_b + S_c } \right)(a - b)^2 + \left( {S_a + S_b } \right)(b - c)^2 + 2S_b (a - b)(b - c)\mathop \ge \limits_{AM - GM} 2\sqrt {\left( {S_a + S_b } \right)\left( {S_b + S_c } \right)} (a - b)(b - c) + 2S_b (a - b)(b - c)$
Do đó bất đẳng thức sẽ được chứng minh nếu ta chứng minh được
$2\sqrt {\left( {S_a + S_b } \right)\left( {S_b + S_c } \right)} + 2S_b - S \ge 0$
Xây dựng tương tự như trên bằng cách tách $a - b = c - b - (c - a)$ và $b - c = b - a - (c - a)$, ta cũng được thêm 2 tiểu chuẩn nữa.
Tiếp tục xây dựng: ta có
$S_b (c - a)^2 = S_b (c - b + b - a)^2 \mathop \ge \limits_{AM - GM} 4S_b (c - b)(b - a)$
và $S_a (b - c)^2 + S_c (a - b)^2 \ge 2\sqrt {S_a .S_c } .(b - a)(c - b)$
Do đó bất đẳng thức sẽ được chứng minh nếu ta chứng minh được $4S_b + 2\sqrt {S_a .S_c } \ge S(c - a)$
Ngoài ra ta còn có $S_a (b - c)^2 + S_b (c - a)^2 + S_c (a - b)^2 \ge 3\sqrt[3]{{S_a S_b S_c (b - c)^2 (c - a)^2 (a - b)^2 }}$

Nên bất đẳng thức sẽ được chứng minh nếu ta chứng minh được
$27 S_a S_b S_c \ge S^3 (a - b)(b - c)(c - a)$
Hệ thống các kết quả trên ta có các tiêu chuẩn sau:
1. $S_a + S_b \ge 0,S_b + S_c \ge 0,2\sqrt {\left( {S_a + S_b } \right)\left( {S_b + S_c } \right)} + 2S_b - S(c - a) \ge 0$
2. $S_a + S_b \ge 0,S_a + S_c \ge 0,2\sqrt {\left( {S_a + S_b } \right)\left( {S_a + S_c } \right)} - 2S_a - S(c - b) \ge 0$
3. $S_c + S_a \ge 0,S_c + S_b \ge 0,2\sqrt {\left( {S_c + S_a } \right)\left( {S_c + S_b } \right)} - 2S_c - S(b - a) \ge 0$
4. $S_a \ge 0,S_c \ge 0,2\sqrt {S_a .S_c } + 4S_b - S(c - a) \ge 0$
5. $S_a \ge 0,S_b \ge 0,S_c \ge 0,2\sqrt {S_b S_c } - S(c - b) \ge 0$
6. $S_a \ge 0,S_b \ge 0,S_c \ge 0,2\sqrt {S_a S_b } - S(b - a) \ge 0$
7. $S_a \ge 0,S_b \ge 0,S_c \ge 0,27S_a S_b S_c - S^3 (a - b)(b - c)(c - a) \ge 0$
Các tiêu chuẩn trên rất tiện để xử lí những bài toán có các hệ số $S_a ;S_b ;S_c$ cồng kềnh (đặc biệt là tiêu chuẩn 1 rất mạnh). Tuy nhiên nếu ta gặp những bài toán rất chặt đến nỗi không thể áp dụng được tiêu chí nào thì có một cách khác là đặt $c = a + x + y$ và $b = a + x (x,y \ge 0)$. Cách làm này giúp ta có thể loại đi a một cách nhanh chóng nhờ cách phân tích trên (bởi c - a = x + y;b - a = x). Hơn nữa ta lại còn có thể làm chặt cho bất đẳng thức nhờ các biến còn thừa lại. Các bài toán áp dụng sau để làm sáng tỏ thêm cho điều này.
Ngoài ra, ta còn có thể chia nhỏ nhiều trường hợp nữa trong $c \ge b \ge a$ để dễ dàng giải quyết bài toán.
V/ Áp dụng vào giải toán.

Có lẽ mọi kĩ thuật cũng xuất phát từ một bài toán nào đó. Và tôi cũng vậy, tôi xin bắt đầu bằng một bài toán khởi đầu cho kĩ thuật này một cách tình cờ:

Bài toán 1. Cho các số thực không âm a,b,c. Chứng minh bất đẳng thức
$a^3 + b^3 + c^3 + 3abc.\dfrac{{a^2 b + b^2 c + c^2 a}}{{ab^2 + bc{}^2 + ca^2 }} \ge ab(a + b) + bc(b + c) + ca(c + a)$
Lời giải.
Nếu$a \ge b \ge c$ thì $a^2 b + b^2 c + c^2 a \ge ab^2 + bc^2 + ca^2$, nên theo bất đẳng thức Schur thì
$a^3 + b^3 + c^3 + 3abc.\dfrac{{a^2 b + b^2 c + c^2 a}}{{ab^2 + bc^2 + ca^2 }} \ge a^3 + b^3 + c^3 + 3abc \ge ab(a + b) + bc(b + c) + ca(c + a)$
Nếu $c \ge b \ge a$ thì bất đẳng thức được viết lại như sau
$a^3 + b^3 + c^3 - 3abc + 3abc.\left( {\dfrac{{a^2 b + b^2 c + c^2 a}}{{ab^2 + bc{}^2 + ca^2 }} - 1} \right) \ge ab(a + b) + bc(b + c) + ca(c + a) - 6abc
\Leftrightarrow \dfrac{1}{2}(a + b + c)\left( {(a - b)^2 + (b - c)^2 + (c - a)^2 } \right) - \dfrac{{3abc(a - b)(b - c)(c - a)}}{{ab^2 + bc{}^2 + ca^2 }} \ge a(b - c)^2 + b(c - a)^2 + c(a - b)^2 \Leftrightarrow \dfrac{1}{2}(a + b - c)(a - b)^2 + \dfrac{1}{2}(b + c - a)(b - c)^2 + \dfrac{1}{2}(c + a - b)(c - a)^2 \ge \dfrac{{3abc(a - b)(b - c)(c - a)}}{{ab^2 + bc{}^2 + ca^2 }}$
Theo tiêu chuẩn 1 thì ta chỉ cần chứng minh $2\sqrt {ac} + c + a - b - \dfrac{{3abc(c - a)}}{{ab^2 + bc{}^2 + ca^2 }} \ge 0$
Quy đồng, rút gọn và nhóm các số hạng lại với nhau ta được bất đẳng thức tương đương là
$2bc^2 \left( {\sqrt {ac} - a} \right) + ab^2 (c - b) + bc^2 (c - b) + a^2 c^2 + a^2 b^2 + a^3 c + 2ab^2 \sqrt {ac} + 2ca^2 \sqrt {ac} + 2a^2 bc \ge 0$
Bất đẳng thức trên đúng do $c \ge b \ge a$
Vậy ta có ta có điều phải chứng minh. Đẳng thức xảy ra khi và chỉ khi ba biến bằng nhua hoặc một trong 3 biến bằng 0 và 2 biến còn lại bằng nhau.


Bài toán 2. (Nguyễn Trọng Thọ) Chứng minh rằng với a,b,c>0 thì:
$\dfrac{{a^3 }}{{2a^2 + b^2 }} + \dfrac{{b^3 }}{{2b^2 + c^2 }} + \dfrac{{c^3 }}{{2c^2 + a^2 }} \ge \dfrac{{a + b + c}}{3}$
Lời giải. để chô gọn ta kí hiệu $\sum$ là tổng cyclic (mỗi tổng gồm 3 số hạng). Bằng cách biến đổi tương đương ta có
$\sum {\dfrac{{a^3 - ab^2 }}{{2a^2 + b^2 }}} \ge 0 \Leftrightarrow \sum {(a^3 - ab^2 )(2b^2 + c^2 )(2c^2 + a^2 ) \ge 0} \Leftrightarrow 3\sum {a^3 b^2 c^2 } + 2\sum {a^3 c^4 } + 2\sum {a^5 b^2 } + \sum {a^5 c{}^2} \ge 4\sum {ab^4 c^2 } + 2\sum {ab^2 c^4 } + 2\sum {a^3 b^4 } \Leftrightarrow 2\sum {\left( {a^5 b^2 + a^3 b^2 c^2 - 2a^4 b^2 c} \right)} + \sum {\left( {a^5 c^2 + a^3 b^2 c^2 - 2a^4 bc^2 } \right)} \ge 2\left( {\sum {a^3 b^4 - a^3 c^4 } } \right) \Leftrightarrow 2\sum {a^3 b^2 (} a - c)^2 + \sum {a^3 c^2 (} a - b)^2 \ge 2(a - b)(b - c)(c - a)\left( {\sum {a^2 b^2 } + \sum {a^2 bc} } \right)$
Bây giờ giả sử $a = m{\rm{ax}}\{ a,b,c{\rm{\} }}$.
Nếu c<b thì bất đẳng thức trên đúng nên ta chỉ phải xét khi $a \ge c \ge b$.
Ta sẽ chứng minh
$2a^3 b^2 (a - c)^2 + 2a^2 c^3 (c - b)^2 + a^3 c^2 (a - b)^2 \ge 2(a - c)(c - b)\left( {a^3 c^2 + a^3 bc + a^3 b^2 } \right)$
Xét 2 trường hợp:
Trường hợp 1. Xét khi $c - b \le a - c$, ta có
$b^2 (a - c)^2 + a^3 c^2 (a - b)^2 \ge 2a^3 b^2 (a - c)(c - b) + 4a^3 c^2 (a - c)(c - b)$
Vì $a^3 b^2 + 2a^3 c^2 \ge a{}^3\left( {c^2 + bc + b^2 } \right)$ nên suy ra điều phải chứng minh.
Trường hợp 2. Xét khi $c - b > a – c$, tương tự như trên ta có
$2a^2 c^3 (c - b)^2 + a^3 c^2 (a - b)^2 \ge 2a^2 c^3 (a - c)(c - b) + 4a^3 c{}^2(a - c)(c - b)$
Vì $a^2 c^3 + 2a^3 c^2 - a^3 \left( {c^2 + bc + b^2 } \right) > a^2 c^2 b + a^3 bc - a^3 \left( {bc + b^2 } \right) = a^3 b(c - b) + a^2 bc(c - a) \ge 0$
Nên ta cũng có điều phải chứng minh.
Vậy bất đẳng thức được chứng minh xong. Đẳng thức xảy ra khi và chỉ khi a=b=c.

Bài toán 3. (VIF) Cho các số thực không âm a,b,c. Chứng minh
$\dfrac{{4a}}{{a + b}} + \dfrac{{4b}}{{b + c}} + \dfrac{{4c}}{{c + a}} + \dfrac{{ab^2 + bc{}^2 + ca^2 + abc}}{{a^2 b + b^2 c + c^2 a + abc}} \ge 7$
Lời giải. theo cách phân tích cơ sở 4 thì bất đẳng thức được viết lại thành
$2\left( {3 - \dfrac{{(a - b)(b - c)(c - a)}}{{(a + b)(b + c)(c + a)}}} \right) + \left( {\dfrac{{ab^2 + bc{}^2 + ca^2 + abc}}{{a^2 b + b^2 c + c^2 a + abc}} - 1} \right) \ge 6$
$\Leftrightarrow \dfrac{{(a - b)(b - c)(c - a)}}{{a^2 b + b^2 c + c^2 a + abc}} - \dfrac{{2(a - b)(b - c)(c - a)}}{{(a + b)(b + c)(c + a)}} \ge 0 \Leftrightarrow \dfrac{{(a - b)(b - c)(c - a)\left[ {(a + b)(b + c)(c + a) - 2\left( {a^2 b + b^2 c + c^2 a + abc} \right)} \right]}}{{\left( {a^2 b + b^2 c + c^2 a + abc} \right)(a + b)(b + c)(c + a)}} \ge 0$
$\Leftrightarrow \dfrac{{\left[ {(a - b)(b - c)(c - a)} \right]^2 }}{{\left( {a^2 b + b^2 c + c^2 a + abc} \right)(a + b)(b + c)(c + a)}} \ge 0$
Bất đẳng thức trên hiển nhiên đúng.
Vậy ta có điều phải chứng minh. Đẳng thức xảy ra khi và chỉ khi a=b=c.

Bài toán 4. (UK TST 2005) cho các số thực dương a,b,c sao cho abc=1. Chứng minh rằng: $\dfrac{{a + 3}}{{(a + 1)^2 }} + \dfrac{{b + 3}}{{(b + 1)^2 }} + \dfrac{{c + 3}}{{(c + 1)^2 }} \ge {\rm{3}}$
Lời giải (VIF). do abc=1 nên đặt $a = \dfrac{y}{x},b = \dfrac{z}{y},c = \dfrac{x}{z}$
Bất đẳng thức trên được viết lại như sau
$\dfrac{{3x^2 + xy}}{{(x + y)^2 }} + \dfrac{{3y^2 + yz}}{{(y + z)^2 }} + \dfrac{{3z^2 + zx}}{{(z + x)^2 }} \ge 3 \Leftrightarrow \dfrac{3}{4}\left[ {\left( {\dfrac{{x - y}}{{x + y}} + 1} \right)^2 + \left( {\dfrac{{y - z}}{{y + z}} + 1} \right)^2 + \left( {\dfrac{{z - x}}{{z + x}} + 1} \right)^2 } \right] + \dfrac{1}{4}\left[ {\dfrac{{(x + y)^2 - (x - y)^2 }}{{(x + y)^2 }} + \dfrac{{(y + z)^2 - (y - z)^2 }}{{(y + z)^2 }} + \dfrac{{(z + x)^2 - (z - x)^2 }}{{(z + x)^2 }}} \right] \ge 3 \Leftrightarrow \left( {\dfrac{{x - y}}{{x + y}}} \right)^2 + \left( {\dfrac{{y - z}}{{y + z}}} \right)^2 + \left( {\dfrac{{z - x}}{{z + x}}} \right)^2 \ge - 3\left( {\dfrac{{x - y}}{{x + y}} + \dfrac{{y - z}}{{y + z}} + \dfrac{{z - x}}{{z + x}}} \right) = \dfrac{{3(x - y)(y - z)(z - x)}}{{(x + y)(y + z)(z + x)}}$
Nếu $(x - y)(y - z)(z - x) \le 0$ thì bđt trên hiển nhiên đúng.
Nếu $(x - y)(y - z)(z - x) \ge 0$ thì
$\left( {\dfrac{{x - y}}{{x + y}}} \right)^2 + \left( {\dfrac{{y - z}}{{y + z}}} \right)^2 + \left( {\dfrac{{z - x}}{{z + x}}} \right)^2 \mathop \ge \limits_{AM - GM} 3\sqrt[3]{{\left( {\dfrac{{(x - y)(y - z)(z - x)}}{{(x + y)(y + z)(z + x)}}} \right)^2 }}$
nên ta chỉ cần chứng minh $\dfrac{{(x - y)(y - z)(z - x)}}{{(x + y)(y + z)(z + x)}} \le 1 \Leftrightarrow 2\left( {x^2 y + y^2 z + z^2 x} \right) \ge 0$ (luôn đúng).
Vậy ta có điều phải chứng minh. Đẳng thức xảy ra khi và chỉ khi a=b=c=1.
Có thể nói hai bài toán trên không cần phải sử dụng đến một tiêu chuẩn nào cả. nhưng mặt khác lại cho thấy được cái lời khi phân tích về $S(a - b)(b - c)(c - a)$.
Và bây giờ chúng ta sẽ thực hiện làm chặt một số bất đẳng thức bằng các biến còn thừa như đã nói ở Lời nói đầu.
Bài toán 5. Cho các số thực không âm a,b,c. Chứng minh bất đẳng thức
$4(a + b + c)^3 \ge 27\left( {ab^2 + bc{}^2 + ca^2 + abc} \right)$
Lời giải.
Nếu $a \ge b \ge c thì ab^2 + bc{}^2 + ca^2 + abc \ge ab^2 + bc{}^2 + ca^2 + abc$ nên
$27\left( {ab^2 + bc{}^2 + ca^2 + abc} \right) \le \dfrac{{27}}{2}\left( {ab^2 + bc{}^2 + ca^2 + ab^2 + bc{}^2 + ca^2 + abc} \right)$
Do đó ta chỉ cần chứng minh $27\left( {ab^2 + bc{}^2 + ca^2 + ab^2 + bc{}^2 + ca^2 + abc} \right) \le 8(a + b + c)^3$
$\Leftrightarrow 8\left( {a^3 + b^3 + c^3 } \right) \ge 3\left( {ab^2 + bc{}^2 + ca^2 + ab^2 + bc{}^2 + ca^2 } \right) + 6abc$
Bất đẳng thức trên hiển nhiên đúng theo bất đẳng thức AM-GM
Nếu $c \ge b \ge a$ thì ta viết bất đẳng thức lại như sau:
$4\sum {a^3 } + 12\sum {a^2 b} - 15\sum {ab^2 - 3abc \ge 0} \Leftrightarrow 4\left( {\sum {a^3 } - 3abc} \right) - \dfrac{3}{2}\left[ {\sum {(ab(a + b))} - 6abc} \right] + \dfrac{{27}}{2}\left( {\sum {a^2 b} - \sum {ab^2 } } \right) \ge 0 \Leftrightarrow 2(a + b + c)\left[ {\sum {(a - b)^2 } } \right] - \dfrac{3}{2}\sum {\left( {a(b - c)^2 } \right)} \ge \dfrac{{27}}{2}(c - b)(b - a)(c - a) \Leftrightarrow (4b + 4c + a)(b - c)^2 + (4c + 4a + b)(c - a)^2 + (4a + 4b + c)(a - b)^2 \ge 27(c - b)(b - a)(c - a) \Leftrightarrow (5a + 5b + 8c)(c - b)^2 + (8a + 5b + 5c)(b - a)^2 + 2(4a + b + 4c)(c - b)(b - a) \ge 27(c - b)(b - a)(c - a)$
Đặt $c = a + x + y,b = a + x$. Bất đẳng thức được viết lại như sau
$y^2 (18a + 8y + 13x) + x^2 (18a + 5y + 10x) + 2(9a + 5x + 4y)xy \ge 27xy(x + y)$
Loại a thì ta chỉ cần chứng minh
$y^2 (8y + 13x) + x^2 (5y + 10x) + 2(5x + 4y)xy \ge 27xy(x + y) \Leftrightarrow 5x^3 + 4y^3 \ge 6x^2 y + 3xy^2$
Ta có $2\left( {x^3 + x^3 + y^3 } \right)\mathop \ge \limits_{AM - GM} 6x^2 y;x^3 + y^3 + y^3 \mathop \ge \limits_{AM - GM} 6xy^2$
Do đó ta có điều phải chứng minh
Bây giờ như đã nói ở phần Xây dựng định lí, ta sẽ làm chặt bất đẳng thức nhờ các biến còn thừa:
$\dfrac{1}{2}.18a\left( {x^2 + y^2 + xy} \right)\mathop \ge \limits_{AM - GM} 9a.\dfrac{3}{4}.(x + y)^2 = \dfrac{{27}}{2}a(c - a)^2$
Như vậy là ta có bất đẳng thức chặt hơn là: với $k = \min \{ a,b,c{\rm{\} }}$ và ${\rm{t = max\{ a}}{\rm{,b}}{\rm{,c\} }}$ thì
$4(a + b + c)^3 \ge 27\left( {ab^2 + bc{}^2 + ca^2 + abc} \right) + \dfrac{{27}}{4}k(t - k)^2$,
Các bạn đừng lo cách làm chặt này chỉ đúng trong một trường hợp mà ta đang xét, bởi trong trường hợp ngược lại ${\rm{a}} \ge {\rm{b}} \ge {\rm{c}}$ thì sau khi đánh giá bất đẳng thức $(a - b)(b - c)(c - a) \le 0 \le - (a - b)(b - c)(c - a)$, công việc còn lại chỉ là vấn đề tương tự.


Bài toán 6. Cho các số thực không âm a,b,c. Chứng minh bất đẳng thức
$a^3 + b^3 + c^3 + 2(a^2 b + b^2 c + c^2 a) \ge 3(ab^2 + bc^2 + ca^2 )$
Lời giải.
Nếu $a \ge b \ge c$ thì $2(a^2 b + b^2 c + c^2 a) \ge 2(ab^2 + bc^2 + ca^2 )$và $a^3 + b^3 + c^3 \ge ab^2 + bc^2 + ca^2$ nên bất đẳng thức hiển nhiên đúng.
Nếu $c \ge b \ge a$ thì bất đẳng thức được viết lại như sau
$(a + b)(a - b)^2 + (b + c)(a - b)^2 + (c + a)(a - b)^2 \ge 5(a - b)(b - c)(c - a) \Leftrightarrow (2a + b + c)(b - a)^2 + (2c + a + b)(c - b)^2 \ge (b - a)(c - b)(3c - 7a)$
Đặt $c = a + x + y,b = a + x$. Bất đẳng thức được viết lại như sau
$x^2 (4a + 2x + y) + y^2 (4a + 3x + 2y) \ge xy( - 4a + 3x + 3y)$
Loại a đi thì ta chỉ cần chứng minh
$x^2 (2x + y) + y^2 (3x + 2y) \ge xy(3x + 3y) \Leftrightarrow 2x^3 + 2y^3 \ge 2x^2 y$
Bất đẳng thức trên đúng do $2x^3 + y^3 \mathop \ge \limits_{AM - GM} 2x^2 y$
Cũng như bài toán trên, ta có thể làm chặt bài toán và thu được:
$a^3 + b^3 + c^3 + 2(a^2 b + b^2 c + c^2 a) \ge 3(ab^2 + bc^2 + ca^2 ) + \dfrac{3}{2}k(t - k)^2$
Bài toán 6. (VIF) cho các số thực dương a,b,c. Chứng minh
$\dfrac{{a^2 + b^2 + c^2 }}{{ab + bc + ca}} + \dfrac{3}{2}.\dfrac{{2\left( {a^2 b + b^2 c + c^2 a} \right) - abc}}{{2\left( {ab^2 + bc{}^2 + ca^2 } \right) - abc}} \ge \dfrac{5}{2}$
Lời giải.
Nếu $a \ge b \ge c thì \dfrac{{2(a^2 b + b^2 c + c^2 a) - abc}}{{2(ab^2 + bc^2 + ca^2 ) - abc}} \ge 1$, nên dễ dàng suy ra điều phải chứng minh
Nếu $c \ge b \ge a$ thì bất đẳng thức được viết lại như sau
$\dfrac{{(a - b)^2 + (b - c)^2 + (c - a)^2 }}{{ab + bc + ca}} \ge \dfrac{{6(a - b)(b - c)(c - a)}}{{2\left( {ab^2 + bc{}^2 + ca^2 } \right) - abc}}$
Mà $\dfrac{{(a - b)^2 + (b - c)^2 + (c - a)^2 }}{{ab + bc + ca}} = \dfrac{{2\left[ {(c - b)^2 + (b - a)^2 + (c - b)(b - a)} \right]}}{{ab + bc + ca}} \ge \dfrac{{6(c - b)(b - a)}}{{ab + bc + ca}}$
Nên ta chỉ cần chứng minh $\dfrac{1}{{ab + bc + ca}} \ge \dfrac{{c - a}}{{2\left( {ab^2 + bc{}^2 + ca^2 } \right) - abc}}$
Quy đồng và rút gọn bất đẳng thức trên thành$c(b - a)(c - a) + 2ca^2 + 3a^2 b \ge 0$
Bất đẳng thức trên hiển nhiên đúng. Vậy ta có điều phải chứng minh. Đẳng thức xảy ra khi và chỉ khi a=b=c.

Bài toán 7. Cho các số thực không âm a,b,c. Chứng minh bất đẳng thức
$\dfrac{{a^3 + b^3 + c^3 }}{3} \ge abc + \dfrac{3}{4}|(a - b)(b - c)(c - a)|$
Lời giải. thật ra ta có thể chứng minh bất đẳng thức mạnh hơn là:
$\dfrac{{a^3 + b^3 + c^3 }}{3} \ge abc + |(a - b)(b - c)(c - a)|$
Không mất tính tổng quát giả sử$c \ge b \ge a$. Bất đẳng thức trên được viết lại như sau
$a^3 + b^3 + c^3 \ge 3abc + 3|(a - b)(b - c)(c - a)| \Leftrightarrow (a + b + c)\left[ {(a - b)^2 + (b - c)^2 + (c - a)^2 } \right] \ge 6|(a - b)(b - c)(c - a)|$
Áp dụng tiêu chuẩn 4, ta cần phải chứng minh $6(a + b + c) - 6(c - a) \ge 0 \Leftrightarrow 12a + 6b \ge 0$
Bất đẳng thức trên hiển nhiên.
Vậy ta có điều phải chứng minh. Đẳng thức xảy ra khi và chỉ khi a=b=c
Thật ra hằng số tốt nhất trong bài toán trên là $\dfrac{1}{3}\sqrt {\dfrac{{3\sqrt 3 + 9}}{{\sqrt 3 - 1}}}$, tức là ta có bất đẳng thức
$\dfrac{{a^3 + b^3 + c^3 }}{3} \ge abc + \dfrac{1}{3}\sqrt {\dfrac{{3\sqrt 3 + 9}}{{\sqrt 3 - 1}}} |(a - b)(b - c)(c - a)|$
Nhưng để giải quyết bài toán này thì cần phải nhờ đến công cụ hàm số, nên không tiện nhắc đến ở đây.


VI/ Bài tập áp dụng.

Qua các ví dụ trên đã phần nào nói lên điểm mạnh của kĩ thuật này, và bây giờ các bạn thử áp dụng phương pháp này để giải quyết các bài toán:

Bài toán 1. Cho các số thực không âm a,b,c sao cho $a^2 + b^2 + c^2 = 3$. Chứng minh
$ab^2 + bc{}^2 + ca^2 \le 2 + abc$
Bài toán 2. Cho các số thực không âm a,b,c sao cho $a^2 + b^2 + c^2 = 1$. Chứng minh
$(a + b + c)(a - b)(b - c)(c - a) \le \dfrac{1}{4}$
Bài toán 3. Cho các số thực không âm a,b,c. chứng minh bất đẳng thức
$\dfrac{a}{{b + c}} + \dfrac{b}{{c + a}} + \dfrac{c}{{a + b}} + \dfrac{{3abc}}{{2\left( {ab^2 + bc{}^2 + ca^2 } \right)}} \ge 2$
Tài liệu tham khảo:

• Sáng tạo bất đẳng thức
Tác giả: Phạm Kim Hùng

• Bất đẳng thức Tìm tòi và Sáng tạo
Tác giả: Võ Quốc Bá Cẩn

• Chuyên đề toán học: Phương pháp chia để trị
Tác giả: Phan Thành Việt

 Các tài liệu từ Internet:
Diễn đàn www.mathlinks.ro
Diễn đàn bất đẳng thức www.vnineqmath.tk